+0  
 
0
2451
1
avatar+1833 

 

Thirty-one books are arranged from left to right in order of increasing prices. The price of each book differs by $2 from that of each adjacent book. For the price of the book at the extreme right, a customer can buy the middle book and a book adjacent to the middle one. Give an expression for the cost of the nth book in terms of n. 

 Aug 4, 2016
 #1
avatar+128521 
+4

Let the price of the first book = P........then we have the following series:

 

an = P + 2(n - 1)    where an  is the price of the  nth  book

 

I assume that you want to know the price of the 31st book??

 

a31  = P + 2(31 - 1)   =  P + 2(30)   =  $ [P + 60]

 

The cost of the middle book - the 16th one - is given by

 

a16 = P + 2(16 -1)  =  P + 2(15)   =  $ [P + 30]

 

However........it is not possible to buy the middle book and an adjacent book for the price of the 31st book

 

The price of the 15th book = $ [ P + 28]    and the price of the 17th book = $ [P + 32]

 

The cost of buying the middle book and the 15th book =  $ [ 2P + 58] which may or may not be greater than the cost of the 31st book.  But, the cost of buying the middle book and the 17th book = $ [2P +  62]  which is definitely greater than the cost of the 31st book. 

 

P. S.-   I hope I have interpreted this correctly  !!!!

 

 

cool cool cool

 Aug 4, 2016

2 Online Users

avatar